Search found 16 matches


still don't get it

Lunar, I still dont understand how the ii) can be taken to mean two numbers multiplied together OR the number multiplied by itself? Isn't that an assumption or am i missing something in wording of this question. Sorry, I know this is a pretty old question

by dwilliams05

Fri Sep 18, 2009 6:11 pm
Forum: Data Sufficiency
Topic: Set of Integer Question
Replies: 10
Views: 3384

maybe I am wrong, but...

when you plug in k=1, you get 1/2. as k increases shouldn't the value of the formula decrease? Hence the answer will never be more than 1/2 so you can eliminate any answer choice that doesn't fit that parameter. Or am I wrong in using that technique?

by dwilliams05

Wed Sep 09, 2009 4:36 pm
Forum: Problem Solving
Topic: GMAT PREP QUESTION w/OA
Replies: 11
Views: 6082

Thanks Stuart. It's similar to a real test question. In fact it was one of the first 10. I don't know why I panicked when I saw this on? Went thru the math several times and couldnt figure out where my mistake was. I also wasted time checking, re-checking, and re-checking again. I finally just moved...

by dwilliams05

Thu Apr 23, 2009 2:36 pm
Forum: Problem Solving
Topic: square roots & decimal
Replies: 2
Views: 1371

square roots & decimal

Ran across a problem similar to this and wanted to know if anyone had a "trick" to answer this type of question? Q: What is the approx value of sqrt[(.81) + (.05) + (.72) / (.003)]? There is no answr solution but I'd like to know a fast method for solving questions types like this (i know ...

by dwilliams05

Thu Apr 23, 2009 2:02 pm
Forum: Problem Solving
Topic: square roots & decimal
Replies: 2
Views: 1371

i obviously have a looong way to go to conquer this quant in order to reach that 700 range. you people are good!! thanks for the help

by dwilliams05

Thu Apr 23, 2009 1:20 pm
Forum: Problem Solving
Topic: 20! is divisible
Replies: 7
Views: 1344

adding exponents is tricky but I believe that if the bases aer different yet the exponents ar the same, then you can simply add the bases together but will need to keep the exponenet the same. in this question 13-14-15-16=58. being that you are dividing by 58, there will be no remainder (58^7/58 = 5...

by dwilliams05

Thu Apr 23, 2009 10:57 am
Forum: Problem Solving
Topic: what is the remainder ?
Replies: 5
Views: 1488

I agree. should be C

(x^1/2)^1/3 = (x^1/3)^1/2. Both are x^1/6

by dwilliams05

Thu Apr 23, 2009 10:48 am
Forum: Problem Solving
Topic: expected answer
Replies: 3
Views: 1131

not sure about this one but...

2N=5A-3 only when A is an odd integer. Thus the first 20 values of N will be from 1, 6, 11, 16, 21, 26, 31, 36, 41, 46, 51, 56....96.

therefore the median will be the avg of two middle terms (since there are an even amount of numbers). 46+51/2 = 48.5

by dwilliams05

Thu Apr 23, 2009 10:38 am
Forum: Problem Solving
Topic: Median
Replies: 4
Views: 1309

yes, please post the OA because I would choose B

by dwilliams05

Thu Apr 09, 2009 12:42 pm
Forum: Sentence Correction
Topic: Univ. of California
Replies: 14
Views: 1980

from 530 to 570 to...

I've been on thsi forum as a visitor for a few months before I actually joined. I've seen some incredible stories and I've seen some similar to my own. I've taken the test twice and will be retaking it on 4/10/09. In my last three practce exams I score 560 (GMAT), 630 (Kaplan), and 570 (GMAT). My sc...

by dwilliams05

Tue Mar 31, 2009 2:14 pm
Forum: I just Beat The GMAT!
Topic: from 530 to 570 to...
Replies: 1
Views: 1485

iwant700plus was dead on in the equation but he set the equation equal to 0. it should be y = 3b(-1/3) + b (taking S1 and S2 together of course). so y = 0 (which is ends up being the y-intercept). please correct me if i am wrong?

by dwilliams05

Tue Mar 31, 2009 1:19 pm
Forum: Data Sufficiency
Topic: Y Intercept Of Line
Replies: 7
Views: 3795

can we assume that the rate is constant? after all it could be drizzling for an hour, rain harder 3 hours later, etc. so just because there's 40% more rain coming does that mean 40% in volume or 40% in terms of time?

I'd go with E but this is a real thought provoker

by dwilliams05

Tue Mar 31, 2009 12:38 pm
Forum: Data Sufficiency
Topic: the continuous rain
Replies: 9
Views: 1832

i concur with iwanta700plus. the answer is C

by dwilliams05

Tue Mar 31, 2009 12:31 pm
Forum: Data Sufficiency
Topic: MGMAT 700-800
Replies: 21
Views: 4381

S1 isn't SUFF. in the equation x < y +1, if x = 3 then y could also equal 3 because 3 < 3 +1 (INSUFF)

similar structure in S2. so my answer would be E
**remember, both x & y are positive numbers**

by dwilliams05

Tue Mar 31, 2009 12:25 pm
Forum: Data Sufficiency
Topic: inequality
Replies: 6
Views: 1688

can you simply just replace x=2y into that equation? because the avg of x is sum of x divided by x. In other words what you're referring to x in the denominator isnt the number of x it's the sum of all the x quantities? Additionally when you take the formula you wrote out 212,000x = 194,000y/x + y, ...

by dwilliams05

Tue Mar 31, 2009 12:10 pm
Forum: Data Sufficiency
Topic: DS
Replies: 3
Views: 1240